LSAT and Law School Admissions Forum

Get expert LSAT preparation and law school admissions advice from PowerScore Test Preparation.

 Administrator
PowerScore Staff
  • PowerScore Staff
  • Posts: 8917
  • Joined: Feb 02, 2011
|
#36321
Complete Question Explanation

Strengthen—PR. The correct answer choice is (D)

In this stimulus, the author attacks Outdoor Sports Company (OSC) for offering customers financial
incentives if they provide the e-mail addresses of their friends. The second half of the stimulus
contains the author’s argument against OSC:
  • Premise: ..... OSC offers incentives that encourage people to exploit their personal relationships for profit, which risks ..... ..... damaging the integrity of those relationships

    Conclusion: ..... Offering such incentives is unethical.
Note that the question stem is a Strengthen—PR, not a Justify—PR because of the presence of
the word “most” in the question stem, which weakens the force required of the correct answer. In
a Strengthen—PR question, the correct answer will provide a premise that, when applied to the
specific situation in the stimulus, helps support the conclusion. Since a principle is by definition
a broad rule (usually conditional in nature), the presence of the Principle indicator serves to
broaden the scope of the question, which requires a more abstract understanding of the underlying
relationships in the argument.

In this problem, you must select a principle that labels as “unethical” the business practice of
encouraging people to engage in the sort of behavior that could damage the integrity of their
friendships. Answer choice (D) contains a statement that is closest to this prephrase.

Answer choice (A): This answer choice seems attractive, because it alludes to the kind of behavior
described in the stimulus. However, by holding that it is unethical for people to exploit their
relationships for profit, this principle misses the point. The author never meant to attack the
customers for providing the e-mail addresses of their friends; she only attacks the business practice
of encouraging them to do so.

Answer choice (B): The author does not argue that gathering information is itself unethical, nor does
she claim that the manner in which such information was gathered is unethical.

Answer choice (C): One major problem with this answer choice is the phrase “deliberately damage.”
Since there is no evidence the OSC purposefully sought to damage the integrity of its customer’s
personal relationships, this principle cannot strengthen the author’s position.

Answer choice (D): This is the correct answer choice. If it is unethical to encourage people to
engage in behavior that could damage the integrity of their personal relationships, and the business
practice described in the stimulus poses such risk, then indeed OSC’s actions should be considered
unethical.

Answer choice (E): Based on the information contained in the stimulus, it seems likely that
providing a friend’s personal information to a company in exchange for a financial reward would
damage the integrity of one’s friendship. However, whether such an action will “almost certainly”
damage the integrity of one’s personal relationship is unclear. Furthermore, answer choice (E) does
not contain a principle establishing that the OSC’s business practice is unethical.
 Nina
  • Posts: 81
  • Joined: Sep 11, 2012
|
#5461
Q16:
i'm still unsure why answer E is incorrect. Because the stimulus can be vulnerable if "providing a friend's personal information to a company in exchange for a financial reward" doesn't damage "the integrity of one's personal relationship with that friend" at all.
and also, answer A seems to having covered the whole reasoning process of the stimulus, why is it incorrect? really confused...:-?

Many thanks!
 Nikki Siclunov
PowerScore Staff
  • PowerScore Staff
  • Posts: 1362
  • Joined: Aug 02, 2011
|
#5469
The author attacks Outdoor Sports Company (OSC) for offering customers financial incentives if they provide the e-mail addresses of their friends. These incentives encourage people to exploit their personal relationships for profit, and are therefore regarded as unethical.

In a Strengthen—PR question, the correct answer will provide a premise that, when applied to the specific situation in the stimulus, helps support the conclusion. Since a principle is by definition a broad rule (usually conditional in nature), the presence of the Principle indicator serves to broaden the scope of the question, which requires a more abstract understanding of the underlying relationships in the argument.

In this problem, you must select a principle that labels as “unethical” the business practice of encouraging people to engage in the sort of behavior that could damage the integrity of their friendships. Answer choice (D) contains a statement that is closest to this prephrase.

Answer choice (A) holds that it is unethical for people to exploit their relationships for profit, which misses the point. The author never meant to attack the customers for providing the e-mail addresses of their friends; she only attacks the business practice of encouraging them to do so.

Answer choice (E) is incorrect for two reasons. First, based on the information contained in the stimulus, it seems likely that providing a friend’s personal information to a company in exchange for a financial reward would damage the integrity of one’s friendship. However, whether such an action will “almost certainly” damage the integrity of one’s personal relationship is unclear. Second, answer choice (E) does not contain a principle establishing that the OSC’s business practice is unethical (which is the conclusion you have to support).

Hope this helps!
 Nina
  • Posts: 81
  • Joined: Sep 11, 2012
|
#5497
Hey Nikki,

Thank you so much for your detailed response. It gives me a clear sense for attacking the Strengthen-PR questions. :)
 Basia W
  • Posts: 108
  • Joined: Jun 19, 2014
|
#15716
Good evening,

my question is in relation to this answer is how to differentiate between c), d) and e) since they all appear to be somewhat similar to me. I often have this difficulty with strengthen questions when the wording becomes quite similar in several answer choices and knowing where on the scale of "strengthen" the right answer lies.

thank you for your help,

best,

Basia
 Luke Haqq
PowerScore Staff
  • PowerScore Staff
  • Posts: 742
  • Joined: Apr 26, 2012
|
#15735
HI basia,

While this question falls into the same category as strengthen questions (and also assumption questions), it should be categorized as a justify-the-conclusion question. Here's how to think of the degree of strength the right answer choices give in these questions:

- Strengthen: It can lend any amount of support to the argument, from 1-100%.

- Justify the conclusion: The right answer targets a gap in the argument. It strengthens the argument--not so much by making it stronger, but rather by letting the conclusion follow from the premises. The right answer is sufficient for the conclusion to follow.

- Assumption: Like justify questions, assumption questions target a hole. However, the right answer on assumption questions is necessary for the argument.

As for question #16 specifically, the conclusion in the stimulus is that it is an unethical business practice to offer the incentives discussed. Since that's the conclusion, we could eliminate (E) because it doesn't talk about unethical business practices.

While it might seem that (C) is stronger than (D), and thus a better answer, I think it actually gets off track. Namely, the word "deliberately" is problematic; if we took (C) to be true, it doesn't necessarily allow the conclusion to follow since we don't know if businesses intend the consequences to relationships. (D), by contrast, bridges a gap in the argument between threatening relationship integrity and unethical behavior, and it does so without presupposing that businesses must intend it for it to be unethical.

Hope that helps!
 Basia W
  • Posts: 108
  • Joined: Jun 19, 2014
|
#15741
Good afternoon!

It does help thank you for your thorough explanation. The question stem had me stumped because of the "most helps" bit because on page 4-18 I remember reading that the stem cannot lessen the degree of justification.

very best,

Basia
 Leela
  • Posts: 63
  • Joined: Apr 13, 2019
|
#64663
If this had been a justify question, rather than a strengthen principle question, would E have been a good answer? How would a justify question stem change our prephrase compared to this strengthen principle question?
 Brook Miscoski
PowerScore Staff
  • PowerScore Staff
  • Posts: 418
  • Joined: Sep 13, 2018
|
#64693
Leela,

Remove the word "most" so that the question asks which principle justifies the reasoning.

(E) would not have been a good answer. If (E) would justify the reasoning, then it would also strengthen the reasoning. (E) does neither because the choice doesn't say anything about ethics. You might not think (E) describes ethical behavior, but you need to restrain yourself to what the choice actually says.

Get the most out of your LSAT Prep Plus subscription.

Analyze and track your performance with our Testing and Analytics Package.